Đến nội dung

PlanBbyFESN

PlanBbyFESN

Đăng ký: 29-04-2015
Offline Đăng nhập: Riêng tư
****-

#675431 $\sqrt{\frac{a}{b}+\frac{b...

Gửi bởi PlanBbyFESN trong 27-03-2017 - 01:00

Bài toán: Với $a,b,c>0$

 

CMR:        $\sqrt{\frac{a}{b}+\frac{b}{c}+\frac{c}{a}-2}+\frac{8abc}{(a+b)(b+c)(c+a)}\geq 2$




#675251 $\prod (a^{2}+b^{2})\leq \frac{1...

Gửi bởi PlanBbyFESN trong 24-03-2017 - 21:35

Bài toán: Cho $a,b,c\geq 0$ thỏa mãn $a+b+c=1$.

 

Chứng minh rằng:  $(a^{2}+b^{2})(b^{2}+c^{2})(c^{2}+a^{2})\leq \frac{1}{32}$




#673541 Max $\sum \frac{1}{a^2+b^2+3}$

Gửi bởi PlanBbyFESN trong 05-03-2017 - 19:15

$\frac{1}{a^2+b^2+3}+\frac{1}{b^2+c^2+3}+\frac{1}{a^2+c^2+3}\leq \frac{3}{2}\Leftrightarrow 2\sum \frac{a^{2}+b^{2}}{a^{2}+b^{2}+3}\geq 3$

 

$2(a^{2}+b^{2})=(a+b)^{2}+(a-b)^{2}$

 

Cauchy Schwarz:

 

$\Rightarrow 2\sum \frac{a^{2}+b^{2}}{a^{2}+b^{2}+3}=\sum \left [ \frac{(a+b)^{2}}{a^{2}+b^{2}+3} +\frac{(a-b)^{2}}{a^{2}+b^{2}+3}\right ]\geq \frac{4(a+b+c)^{2}+4\left \{(a-c)^{2} ;(b-a)^{2};(c-b)^{2} \right \}}{2(a^{2}+b^{2}+c^{2})+27}$

 

Mà $27=2(a+b+c)^{2}$. Quy đồng BĐT dưới dang thuần nhất ta được:

 

$\frac{4(a+b+c)^{2}+4(a-c)^{2}}{2(a^{2}+b^{2}+c^{2})+27}\geq 3\Leftrightarrow \frac{4(a+b+c)^{2}+4(a-c)^{2}}{2(a^{2}+b^{2}+c^{2})+2(a+b+c)^{2}}\geq 3$

 

$\Leftrightarrow (a-b)(b-c)\geq 0$

Tương tự ta cũng có: $\Leftrightarrow \begin{bmatrix} (b-c)(c-a)\geq 0 & \\ (c-a)(a-b)\geq 0 & \end{bmatrix}$

 

$\left [(a-b)(b-c)(c-a) \right ]^{2}\geq 0\Rightarrow$ Tồn tại 1 BĐT đúng! $\blacksquare$

 



#672209 Cho ba số a,b,c>0. CMR:

Gửi bởi PlanBbyFESN trong 20-02-2017 - 20:28

Cho ba số a,b,c>0. CMR: 

$ \frac{2(a^{5}+b^{5}+c^{5}+a^{2}+b^{2}+c^{2})}{a^{3}+b^{3}+c^{3}}+\frac{9}{(a+b+c)^{2}} \geq 5 $

 

$\frac{2(a^{5}+b^{5}+c^{5}+a^{2}+b^{2}+c^{2})}{a^{3}+b^{3}+c^{3}}+\frac{9}{(a+b+c)^{2}}= \frac{(a^{5}+b^{5}+c^{5})}{a^{3}+b^{3}+c^{3}}+\frac{(a^{5}+b^{5}+c^{5})}{a^{3}+b^{3}+c^{3}}+\frac{(a^{2}+b^{2}+c^{2})}{a^{3}+b^{3}+c^{3}}+\frac{(a^{2}+b^{2}+c^{2})}{a^{3}+b^{3}+c^{3}}+\frac{9}{(a+b+c)^{2}}\geq 5\sqrt[5]{\frac{9(a^{5}+b^{5}+c^{5})^{2}(a^{2}+b^{2}+c^{2})^{2}}{((a^{3}+b^{3}+c^{3})^{4})(a+b+c)^{2}}}$

 

Cần CM: $3(a^{5}+b^{5}+c^{5})(a^{2}+b^{2}+c^{2})\geq (a^{3}+b^{3}+c^{3})^{2}(a+b+c)$

 

Hiển nhiên từ 2 BĐT sau: 

 

$\left\{\begin{matrix} (a^{5}+b^{5}+c^{5})(a+b+c)\geq (a^{3}+b^{3}+c^{3})^{2} & \\ 3(a^{2}+b^{2}+c^{2})\geq (a+b+c)^{2} & \end{matrix}\right.$




#672061 $(a^2+2bc)(b^2+2ca)(c^2+2ab)\geq abc(a+2b)(b+2c)(c+2a)$

Gửi bởi PlanBbyFESN trong 19-02-2017 - 10:07

Cho $a,b,c>0$.  Chứng minh rằng:

$(a^2+2bc)(b^2+2ca)(c^2+2ab)\geq abc(a+2b)(b+2c)(c+2a)$

 

 

$\left\{\begin{matrix} (a^{2}+2bc)(1+\frac{2b}{c})\geq (a+2b)^{2} & \\ (c^{2}+2ab)(1+\frac{2b}{a})\geq (c+2b)^{2} & \end{matrix}\right.\Rightarrow\left\{\begin{matrix} (a^{2}+2bc)^2(1+\frac{2b}{c})^2\geq (a+2b)^{4} & \\ (c^{2}+2ab)(1+\frac{2b}{a})\geq (c+2b)^{2} & \end{matrix}\right.$

 

$\Rightarrow (a^{2}+2bc)^{2}(c^{2}+2ab)\geq ac^{2}(a+2b)^{3}$

 

Tương tự 2 BĐT còn lại nhân lại hạ bậc ta có ĐPCM :)




#672057 $54x^3 + 1=y^3$

Gửi bởi PlanBbyFESN trong 19-02-2017 - 09:48

Xin trích lại lời giải của anh nguyenta98:

 

 

 

Giải như sau:
Đặt $a=2x^3$ khi ấy $27a+1=y^3,a=2x^3 \Rightarrow a(27a+1)=2(xy)^3=2t^3$
Suy ra $2a(54a+2)=(2t)^3=k^3$ suy ra $u(27u+2)=k^3 \Rightarrow 9u(3.(9u)+2)=9k^3$
Do đó đặt $v=9v$ khi ấy $v(3v+2)=9k^3 \Rightarrow 3v(3v+2)=(3k)^3=m^3$
Lúc này phương trình là $9v^2+6v=m^3 \Rightarrow (3v+1)^2=m^3+1=(m+1)(m^2-m+1)$

Vì $gcd(m+1,m^2-m+1)=1,3$ mà $3v+1 \not \vdots 3$ nên $gcd(m+1,m^2-m+1)=1$ do đó $m^2-m+1=l^2$ giải phương trình nghiệm nguyên này thu được $m=0$ do đó $v=0$

Đưa về quá trình đặt ẩn ban đầu thu được $x=0,y=1$

 

 

 




#670308 C/m $\frac{x}{\sqrt[3]{yz}}+...

Gửi bởi PlanBbyFESN trong 29-01-2017 - 10:19

cho các số thực dương x,y,z thỏa mãn $x^{2}+y^{2}+z^{2}=3$ 

C/m $\frac{x}{\sqrt[3]{yz}}+\frac{y}{\sqrt[3]{xz}}+\frac{z}{\sqrt[3]{yx}}\geq xy+yz+xz$

 

$\frac{x}{\sqrt[3]{yz}}+\frac{y}{\sqrt[3]{xz}}+\frac{z}{\sqrt[3]{yx}}=\frac{x^{2}}{x\sqrt[3]{yz}}+\frac{y^{2}}{y\sqrt[3]{xz}}+\frac{z^{2}}{z\sqrt[3]{yx}}\geq \frac{(x+y+z)^{2}}{\sqrt[3]{xyz}(\sqrt[3]{x^{2}}+\sqrt[3]{y^{2}}+\sqrt[3]{z^{2}})}\geq (xy+yz+zx)$

 

Hiển nhiên vì $xyz\leq 1;\sum \sqrt[3]{x^{2}}\leq \sum \frac{(x^{2}+1+1)}{3}=3$




#670147 $\frac{b^2}{a}+\frac{c^2}{b...

Gửi bởi PlanBbyFESN trong 27-01-2017 - 16:10

Em cảm ơn chị!

P/s:Giả thiết cho mà mình không sử dụng vẫn được hả chị? Em tưởng cho cái gì thì phải dùng cái đó chứ!

 

Khi xét dấu "=" vẫn phải dùng giả thiết chứ em :) Dấu "=" của BĐT chị là $a=b=c$ kết hợp giả thiết $abc=1$ và $a,b,c$ dương ta có dấu bằng là $a=b=c=1$




#670092 $\frac{b^2}{a}+\frac{c^2}{b...

Gửi bởi PlanBbyFESN trong 27-01-2017 - 08:12

Chị ơi, sao ra được cái này zạ? Em ko hiểu lắm! Mà hình như chị ko sử dụng gt abc=1?

P/s: Câu ns của Bill? Ý chị là cái chữ kí của em ý hả? Em thấy nó hay mà!

 

$(a+b+c)^{2}\geq 3(ab+bc+ca)\Leftrightarrow \frac{1}{2}\left [(a-b)^{2}+(b-c)^{2}+(c-a)^{2} \right ]\geq 0\Rightarrow a+b+c\geq \sqrt{3(ab+bc+ca)}$

 

Bài này chỉ cần $a,b,c>0$ hoặc có cách khác sử dụng $abc=1$ còn cách chị thì giả thiết này cho đẹp thôi!

 




#669982 $\frac{b^2}{a}+\frac{c^2}{b...

Gửi bởi PlanBbyFESN trong 26-01-2017 - 10:15

Cho ba số dương a, b, c thỏa abc=1. Chứng minh: $\frac{b^2}{a}+\frac{c^2}{b}+\frac{a^2}{c}+\frac{9}{2(ab+bc+ca)}$>=$\frac{9}{2}$

 

$\frac{b^2}{a}+\frac{c^2}{b}+\frac{a^2}{c}+\frac{9}{2(ab+bc+ca)}\geq a+b+c+\frac{9}{2(ab+bc+ca)}\geq \sqrt{3(ab+bc+ca)}+\frac{9}{2(ab+bc+ca)}=\frac{\sqrt{3(ab+bc+ca)}}{2}+\frac{\sqrt{3(ab+bc+ca)}}{2}+\frac{9}{2(ab+bc+ca)}\geq \frac{9}{2}$      (Am-Gm)

 




#669979 $(a^2+b^2+c^2).(\frac{1}{a^2}+\frac{1...

Gửi bởi PlanBbyFESN trong 26-01-2017 - 09:58

Cho a,b,c là độ dài 3 cạnh của tam giác không nhọn .CHứng minh rằng
$(a^2+b^2+c^2).(\frac{1}{a^2}+\frac{1}{b^2}+\frac{1}{c^2})\geq 10$

 

$\Leftrightarrow \sum \left (\frac{a^{2}}{b^{2}}+\frac{b^{2}}{a^{2}} \right )\geq 7$

 

Giả sử a là cảnh lớn nhất: $\Rightarrow a\geq \sqrt{b^{2}+c^{2}}\geq \sqrt{2bc}$   (Tam giác không nhọn)

 

$\Rightarrow (\frac{b^{2}}{c^{2}}+\frac{c^{2}}{b^{2}})+(\frac{a^{2}}{4c^{2}}+\frac{c^{2}}{a^{2}})+(\frac{a^{2}}{4b^{2}}+\frac{b^{2}}{a^{2}})+\frac{3}{4}(\frac{a^{2}}{b^{2}}+\frac{a^{2}}{c^{2}})\geq 2+1+1+3=7$  (Am-Gm)




#669846 $x^4 + x + \frac{1}{2} > 0$

Gửi bởi PlanBbyFESN trong 25-01-2017 - 11:37

Cho số thực $0 < t < 1.$ Chứng minh rằng

\[3+36t^2+6t^3 \geqslant 20t(t^3+1).\]

 

$3+36t^2+6t^3 \geqslant 20t(t^3+1)\Leftrightarrow \left [2+26t^{2}+6t^{3}-20t^{4}-14t \right ]+\left [ 10t^{2}+1-6t \right ]\geq 0$

 

$\Leftrightarrow 2(1-t)(10t^{3}+7t^{2}-6t+1)+t^{2}+(3t-1)^{2}\geq 0$

 

Đúng vì $10t^{3}+7t^{2}+\frac{1}{3}+\frac{1}{3}+\frac{1}{3}\geq 5\sqrt[5]{\frac{70}{27}}.t\approx 6.04946t> 6t> 0$




#669456 1, Với x, y >0. Tìm giá trị nhỏ nhất của biểu thức: $Q=\sqrt...

Gửi bởi PlanBbyFESN trong 22-01-2017 - 21:46

2, Với mọi số dương a;b;c. Chứng minh: $\frac{a^2}{a+b}+\frac{b^2}{b+c}+\frac{c^2}{c+a}+\frac{1}{2}(\sqrt{ab}+\sqrt{bc}+\sqrt{ca})\geq a+b+c$

 

Trông có vẻ phức tạp nhưng :

 

$\frac{a^2}{a+b}+\frac{b^2}{b+c}+\frac{c^2}{c+a}+\frac{1}{2}(\sqrt{ab}+\sqrt{bc}+\sqrt{ca})\geq a+b+c$

 

$\Leftrightarrow \frac{1}{2}(\sqrt{ab}+\sqrt{bc}+\sqrt{ca})\geq \frac{ab}{a+b}+\frac{bc}{b+c}+\frac{ca}{c+a}$

 

$\frac{ab}{a+b}\leq \frac{ab}{2\sqrt{ab}}=\frac{1}{2}\sqrt{ab}$.............




#669449 1, Với x, y >0. Tìm giá trị nhỏ nhất của biểu thức: $Q=\sqrt...

Gửi bởi PlanBbyFESN trong 22-01-2017 - 21:35

1, Với x, y >0. Tìm giá trị nhỏ nhất của biểu thức: $Q=\sqrt{\frac{x^3}{x^3+8y^3}}+\sqrt{\frac{4y^3}{y^3+(x+y)^3}}$
 

 

Am-Gm:

 

$\sqrt{\frac{x^3}{x^3+8y^3}}=\sqrt{\frac{1}{1+(\frac{2y}{x})^{3}}}=\frac{1}{\sqrt{(1+\frac{2y}{x})(\frac{4y^{2}}{x^{2}}-\frac{2y}{x}+1)}}\geq \frac{2}{\frac{4y^{2}}{x^{2}}+2}=\frac{1}{\frac{2y^{2}}{x^{2}}+1}$

 

$\sqrt{\frac{4y^3}{y^3+(x+y)^3}}=\sqrt{\frac{4}{1+(\frac{x}{y}+1)^{3}}}=\frac{2}{(\sqrt{1+\frac{x}{y}+1)((\frac{x}{y}+1)^{2}-\frac{x}{y})}}\geq \frac{4}{(\frac{x}{y}+1)^{2}+1}$

 

Đặt $\frac{x}{y}=a\rightarrow \frac{y}{x}=\frac{1}{a}$

 

CM: $\frac{1}{\frac{2}{a^{2}}+1}+\frac{4}{(a+1)^{2}+2}\geq 1\Leftrightarrow 2(a-1)^{2}\geq 0$

 

$\Rightarrow \sqrt{\frac{x^3}{x^3+8y^3}}+\sqrt{\frac{4y^3}{y^3+(x+y)^3}}\geq 1$




#669420 $\frac{1}{2+a^2b}+\frac{1}{2+b^2c}+\frac{1}{2+c^2a}\...

Gửi bởi PlanBbyFESN trong 22-01-2017 - 20:13

Cho a,b,c>0, $\sum a=3$

CMR $\frac{1}{2+a^2b}+\frac{1}{2+b^2c}+\frac{1}{2+c^2a}\geq 1$

 

$\frac{1}{2+a^2b}+\frac{1}{2+b^2c}+\frac{1}{2+c^2a}\geq 1$

 

 

$\frac{a^2b}{2+a^2b}+\frac{b^2c}{2+b^2c}+\frac{c^2a}{2+c^2a}\leq 1$

 

Am-Gm:

 

$\frac{a^2b}{2+a^2b}=\frac{a^2b}{1+1+a^2b}\leq \frac{a^{2}b}{3\sqrt[3]{a^{2}b}}=\frac{1}{3}\sqrt[3]{a^{4}b^{2}}=\frac{1}{3}\sqrt[3]{a^2.ab.ab}\leq \frac{1}{9}(a^2+2ab)$

Tương tự ...........

 

$\Rightarrow \frac{a^2b}{2+a^2b}+\frac{b^2c}{2+b^2c}+\frac{c^2a}{2+c^2a}\leq \frac{1}{9}(a+b+c)^{2}=1$       (ĐPCM)